Q5

 
dlclay7
Thanks Received: 0
Vinny Gambini
Vinny Gambini
 
Posts: 3
Joined: January 11th, 2010
 
 
 

Q5

by dlclay7 Mon Jun 14, 2010 8:53 pm

If Q is more popular than L than each of the following must be true of the ranking except:

a) H is first
b) L is fourth
c) V is not fourth
d) J is not third
e) Q is third

I need help with putting this new rule change into my diagram
User avatar
 
ManhattanPrepLSAT1
Thanks Received: 1909
Atticus Finch
Atticus Finch
 
Posts: 2851
Joined: October 07th, 2009
 
 
 

Re: PT 33, S4, G1, Q 5

by ManhattanPrepLSAT1 Mon Jun 14, 2010 10:13 pm

I've attached a slide with both the original tree as well as an adjusted tree to help you work through the answer choices. It should be apparent that answer choice (B) is the only answer choice that could be false.

If you need more help with this one let me know!
Attachments
PT33, S4, G1, Q5 - Television Program Popularity - ManhattanLSAT.pdf
(40.33 KiB) Downloaded 262 times